Diners at several restaurants were surveyed to see what day of the week they prefer to go out to eat. The following table gives some of the data collected by the restaurants.


Restaurant Number of Customers

Who Prefer Saturdays Number of

Customers Surveyed

A 26 30

B 35 72

C 38 143

D 15 64

E 63 74

If each restaurant selects one customer from the group of people surveyed, selecting a customer who prefers to go out to eat on Saturday is an unlikely event for
.


If each restaurant selects one customer from the group of people surveyed, selecting a customer who prefers to go out to eat on Saturday is a likely event for
.


If each restaurant selects one customer from the group of people surveyed, selecting a customer who prefers to go out to eat on Saturday is neither a likely event nor an unlikely event for

Answers

Answer 1

If each restaurant selects one customer from the group of people surveyed, selecting a customer who prefers to go out to eat on Saturday is an unlikely event for Restaurants C and D.

If each restaurant selects one customer from the group of people surveyed, selecting a customer who prefers to go out to eat on Saturday is a likely event for Restaurants A and E.

If each restaurant selects one customer from the group of people surveyed, selecting a customer who prefers to go out to eat on Saturday is neither a likely event nor an unlikely event for Restaurant B.

How to find the likelihood ?

Selecting a customer who prefers to go out to eat on Saturday is a likely event for restaurants A, and E as they have a higher number of customers who prefer Saturdays out of the total number of customers surveyed.

Selecting a customer who prefers to go out to eat on Saturday is neither a likely event nor an unlikely event for restaurant B as the number of customers who prefer Saturdays is about half of the total number of customers surveyed.

Find out more on customers at https://brainly.com/question/30265021

#SPJ1


Related Questions

A student answers a multiple choice examination question that offers four possible answers. Suppose the probability that the student knows the answer to the question is .8 and the probability that the student will guess is .2. Assume that if the student guesses, the probability of selecting the correct answer is .25. If the student correctly answers a question, what is the probability that the student really knew the correct answer?

Answers

If the student correctly answers a question, the probability that the student really knew the correct answer is  0.941.

The probability that the student knows the answer to the question and correctly answers it is 0.8 x 1 = 0.8. The probability that the student guesses and correctly answers the question is 0.2 x 0.25 = 0.05. The probability that the student correctly answers the question is 0.8 + 0.05 = 0.85.

The probability that the student really knew the correct answer given that they correctly answered the question is 0.8 / 0.85 = 0.941. Therefore, the probability that the student really knew the correct answer is approximately 94.1%.

Know more about probability here:

https://brainly.com/question/30034780

#SPJ11

The city of Irvine reported that approximately 75% of residents are over the age of 60. Let X be the number of Irvine residents over the age of 60.From a random sample of 500 Irvine residents, 350 were over the age of 60.What is the sampling distribution of the sample proportion for the sample size of 500?Using the distribution of X from above, what is the probability that at most 350 of the 500 Irvine residents selected will be over the age of 60?What is the probability that at least 350 of the 500 residents in the sample were over the age of 60?What is the probability that between 400 and 475 of the residents were over the age of 60?

Answers

The probability that at most 350 of the 500 Irvine residents selected will be over the age of 60 is 0.9292. The probability that at least 350 of the 500 residents in the sample were over the age of 60 is 0.0708. The probability that between 400 and 475 of the residents were over the age of 60 is 5.88.


The probability that at most 350 of the 500 Irvine residents selected will be over the age of 60 can be found by calculating the z-score for 350 and finding the corresponding probability from a normal distribution table. The z-score for 350 is (350-375)/17 = -1.47. The corresponding probability from a normal distribution table is 0.0708.


The sampling distribution of the sample proportion for the sample size of 500 is a normal distribution with a mean of 0.75 and a standard deviation of  [tex]√[(0.75)(0.25)/500] = 0.017.[/tex]


The probability that at least 350 of the 500 residents in the sample were over the age of 60 can be found by calculating the z-score for 350 and finding the corresponding probability from a normal distribution table. The z-score for 350 is (350-375)/17 = -1.47. The corresponding probability from a normal distribution table is 1 - 0.0708 = 0.9292.



The probability that between 400 and 475 of the residents were over the age of 60 can be found by calculating the z-scores for 400 and 475 and finding the corresponding probabilities from a normal distribution table. The z-score for 400 is (400-375)/17 = 1.47 and the z-score for 475 is (475-375)/17 = 5.88.

The corresponding probabilities from a normal distribution table are 0.9292 and 1.0000, respectively. The probability that between 400 and 475 of the residents were over the age of 60 is 1.0000 - 0.9292 = 0.0708.

Know more about probability here:

https://brainly.com/question/30034780

#SPJ11

Eleven percent of the products produced by an industrial process over the past several months fail to conform to specifications. The company modifies the process attempting to reduce the rate of noncomforties. In a trial run, the modified process produces 16 noncomforting items out of 300 produced. Construct and interpret a 95% ci for the proportion of noncomforming items

Answers

The 95% confidence interval for the proportion of nonconforming items is (0.093, 0.250). This indicates that there is a 95% chance that the true proportion of nonconforming items lies between 9.3% and 25%.

To calculate the 95% confidence interval for the proportion of nonconforming items, we first calculate the sample proportion p of nonconforming items: p = 16/300 = 0.053.

Next, we calculate the standard error of the sample proportion, which is SE = √(p(1-p)/n) = √(0.053(1-0.053)/300) = 0.01.

Finally, we calculate the lower and upper limits of the 95% confidence interval for the proportion of nonconforming items by subtracting and adding 1.96 x SE to the sample proportion, respectively. This gives us the confidence interval of (0.093, 0.250).

Learn more about interval here

https://brainly.com/question/14641200

#SPJ1

5+5+5=15pts (Quadrics) Let Q=1 be a quadric surface in 3-dimensional affine Euclidean space R^3. (See the list of reduced quadrics given in Lecture 5). Determine which of these quadrics are - regular 2-surfaces; - ruled 2-surfaces. On p.271 of the book, M. Audin mentions that all quadrics appear as ruled surfaces if one allows lines to be imaginary. What does she mean by this statement?

Answers

In 3-dimensional affine Euclidean space R³, a regular 2-surface is a surface that has a well-defined tangent plane at each point on the surface, and a ruled 2-surface is a surface that can be generated by moving a straight line (the generator) along a curve (the directrix) on the surface.

What are quadric surfaces like?

For the quadric surfaces, we have:

Ellipsoid: Regular 2-surfaceHyperboloid of one sheet: Regular 2-surfaceHyperboloid of two sheets: Regular 2-surfaceCone: Not a regular 2-surface (the vertex is a singular point)Elliptic paraboloid: Ruled 2-surfaceHyperbolic paraboloid: Ruled 2-surfaceCylinder: Ruled 2-surfaceSphere: Not a regular 2-surface (the center is a singular point)

Regarding the statement by M. Audin, she means that if we allow lines to be imaginary, then any quadric surface can be generated by moving a straight line (even if it is an imaginary line) along a curve on the surface.

In other words, every quadric surface can be considered a ruled surface if we allow imaginary lines. This is a consequence of the fact that any two points on a quadric surface can be connected by at least one real or imaginary line.

learn more about quadric surfaces: https://brainly.com/question/12955029

#SPJ1

A small publishing company is releasing a new book. The production costs will include a one-time fixed cost for editing and an additional cost for each book

printed. The total production cost C (in dollars) is given by the function C = 18. 95N+750, where N is the number of books.

The total revenue earned (in dollars) from selling the books is given by the function R = 34. 60N.

Let P be the profit made (in dollars). Write an equation relating P to N. Simplify your answer as much as possible

Answers

If The total revenue earned (in dollars) from selling the books is given by the function R-34.60N. the equation relating P to N is P = 18.95N - 750.

Profit made (P) can be calculated by subtracting the total production cost (C) from the total revenue earned (R), so:

P = R - C

P = 34.60N - (18.95N + 750)

P = 34.60N - 18.95N - 750

P = 15.65N - 750

Therefore, the equation relating P to N is P = 18.95N - 750.

The equation shows that the profit made by the company is a linear function of the number of books printed. The slope of the line is the revenue per book (34.60 dollars), minus the cost per book (18.95 dollars), which is 18.95 dollars.

The intercept of the line is the fixed cost for editing (750 dollars). The equation can be used to estimate the profit for any given number of books printed, and to determine the break-even point, which is the number of books that need to be sold to cover the total production cost.

Learn more about total revenue:

https://brainly.com/question/13000391

#SPJ4

If 45% of a number is 153 and 10% of the same number is 34, find 55% of that number

Answers

Answer:

187

Step-by-step explanation:

We can calculate the number by doing 153/.45 to give us 340.

We can also do 34/.1 to give us 340.

Since we got 340 as our answer for the first two numbers, convert 55% to .55 and multiply by 340 to get 187.

The coordinates of the points A and B are (0, 6) and (8, 0) respectively. (i) Find the equation of the line passing through A and B. Given that the line y = x + 1 cuts the line AB at the point MÄUK (ii) the coordinates of M, (iii) the equation of the line which passes through M and is parallel to the x-axis, (iv) the equation of the line which passes through M and is parallel to the y-axis.

Answers

Therefore, the equation of this line is: x = 8/7.

What is equation?

An equation is a mathematical statement that shows that two expressions are equal. It contains one or more variables, and the goal is to solve for the value(s) of the variable(s) that make the equation true.

Given by the question.

To find the equation of the line passing through points A and B, we need to determine the slope and the y-intercept of the line.

The slope of the line can be found using the formula:

slope = (change in y) / (change in x)

Using the coordinates of A and B, we have:

slope = (0 - 6) / (8 - 0) = -6/8 = -3/4

The y-intercept of the line can be found by substituting the coordinates of point A and the slope into the slope-intercept form of the equation of a line:

y = mx + b

where m is the slope and b are the y-intercept.

Using the coordinates of point, A and the slope we just calculated, we have:

6 = (-3/4) (0) + b

b = 6

Therefore, the equation of the line passing through points A and B is:

y = -3/4 x + 6

(ii) To find the coordinates of point M where the line y = x + 1 intersects the line AB, we need to solve the system of equations:

y = -3/4 x + 6 (equation of line AB)

y = x + 1 (equation of line y = x + 1)

Substituting y = x + 1 into the equation of line AB, we have:

x + 1 = -3/4 x + 6

Solving for x, we have:

x = 8/7

Substituting x = 8/7 into the equation of line y = x + 1, we have:

y = 8/7 + 1 = 15/7

Therefore, the coordinates of point M are:

M (8/7, 15/7)

(iii) The line passing through point M and parallel to the x-axis is a horizontal line with equation y = c, where c is the y-coordinate of point M. Therefore, the equation of this line is:

y = 15/7

(iv) The line passing through point M and parallel to the y-axis is a vertical line with equation x = c, where c is the x-coordinate of point M.

To learn more about variable:

https://brainly.com/question/17344045

#SPJ9

Jermaine spent $204 dollars on

shirts for his 21 employees while he

was on his vacation. Large shirts

were $12 and small shirts were $8.

How many larges did he buy?

Answers

Jermaine bought 9 large shirts for $12, and 12 small shirts for $8 for his 21 employees.

Let's represent the number of large shirts Jermaine bought as "L" and the number of small shirts as "S". We can set up a system of equations based on the information given:

L + S = 21 (equation 1, the total number of shirts is 21)

12L + 8S = 204 (equation 2, the total cost of the shirts is $204)

To solve for L, we need to eliminate S. We can do this by multiplying equation 1 by 8 and subtracting it from equation 2:

12L + 8S = 204

8L + 8S = 168 (multiply equation 1 by 8)

4L = 36

Dividing both sides by 4, we get:

L = 9

Therefore, Jermaine bought 9 large shirts for his 21 employees. We can find the number of small shirts by substituting L = 9 into equation 1:

L + S = 21

9 + S = 21

S = 12

So, by using linear equation system we find that Jermaine bought 9 large shirts and 12 small shirts for his 21 employees.

To know more about Purchase:

https://brainly.com/question/18649419

#SPJ4

Quality Progress, February 2005, reports on improvements in customer satisfaction and loyalty made by Bank of America. A key measure of customer satisfaction is the response (on a scale from 1 to 10) to the question: "Considering all the business you do with Bank of America, what is your overall satisfaction with Bank of America?" Here, a response of 9 or 10 represents "customer delight." Suppose that the survey selected 350 customers. Assume that 48% of Bank of America customers would currently express customer delight. That is, assume p = .48.
Find the probability that the sample proportion obtained from the sample of 350 Bank of America customers would be within three percentage points of the population proportion. That is, find P(.45 < Picture < .51). (Round your answer to 4 decimal places. Do not round intermediate values. Round z-value to 2 decimal places.) P(.45 < Picture < .51) .7372
Find the probability that the sample proportion obtained from the sample of 350 Bank of America customers would be within six percentage points of the population proportion. That is, find P(.42 < Picture < .54). (Round your answer to 4 decimal places. Do not round intermediate values. Round z-value to 2 decimal places.) P(.42 < Picture < .54)

Answers

From the given data, the probability that the sample proportion is between 0.45 and 0.51 is approximately 0.7372 and between 0.42 and 0.54 is 0.9772.

To solve this problem, we can use the central limit theorem, which states that the distribution of sample proportions will be approximately normal for large sample sizes.

Given that the population proportion is p = 0.48 and the sample size is n = 350, we can calculate the standard error of the sample proportion as:

SE = √(p × (1 - p) / n) = √(0.48 × 0.52 / 350) = 0.025

We want to find the probability that the sample proportion is within three percentage points of the population proportion, or in other words, between 0.45 and 0.51. To do this, we can standardize the sample proportion using the standard error:

z = (P - p) / SE = (0.45 - 0.48) / 0.025 = -1.2

z = (P - p) / SE = (0.51 - 0.48) / 0.025 = 1.2

Using a standard normal distribution table or calculator, we can find the area under the curve between these two z-values, which represents the probability that the sample proportion is within three percentage points of the population proportion:

P(-1.2 < z < 1.2) = 0.7372

To find the probability that the sample proportion is within six percentage points of the population proportion, or between 0.42 and 0.54, we can use the same approach:

z = (P - p) / SE = (0.42 - 0.48) / 0.025 = -2.4

z = (P - p) / SE = (0.54 - 0.48) / 0.025 = 2.4

Again, using a standard normal distribution table or calculator, we can find the area under the curve between these two z-values:

P(-2.4 < z < 2.4) = 0.9772

Learn more about probability here: brainly.com/question/25839839

#SPJ1

Somebody please help me with my homework

Answers

The missing angle measures are 50 degrees, 100 degrees, and 80 degrees.

x = 50, we can substitute this value into the expressions for the other two angles:

2x = 2(50) = 100

x + 30 = 50 + 30 = 80

What are angles?

When two rays are united at a common point, an angle is created. The two rays are referred to as the arms of the angle, while the common point is referred to as the node or vertex. The symbol stands for the angle. Angle is a derivative of the Latin word "Angulus."

The construction of an angle is a type of geometric shape made by connecting two rays at their termini. Three letters that make up the shape of the angle can alternatively be used to symbolize the angle, with the middle letter indicating the location of the angle (i.e.its vertex).

From the question:

The total of the measures of the angles in any triangle is always 180 degrees, as shown by the characteristics of triangle angles.

This knowledge allows us to construct an equation to account for the missing angle measurements:

x + 2x + 30 = 180

Combining like terms, we get:

3x + 30 = 180

Subtracting 30 from both sides, we get:

3x = 150

Dividing both sides by 3, we get:

x = 50

Knowing that x = 50, we can change the formulas for the remaining two angles to reflect this value:

2x = 2(50) = 100

x + 30 = 50 + 30 = 80

As a result, the missing angle measurements are 50, 100, and 80 degrees.

to know more about angle visit:

https://brainly.com/question/28451077

#SPJ1

What is -5/6 divided -1/3? answers A -5/18 B -5/2 C 5/2 D 5/18

Answers

the numerators are both negative, the answer will be a negative number. The numerator of the first fraction (-5) divided by the numerator of the second fraction (5) is -1. Multiply this answer by the common denominator (18) to get the final answer: -5/2.

To solve this fraction division problem, first convert the fractions to have a common denominator. To do this, multiply the denominator of the first fraction (-1/3) by the denominator of the second fraction (6), and the denominator of the second fraction (6) by the denominator of the first fraction (-1/3). This will change the fractions to -5/18 and 5/18, respectively.

Next, divide the numerators of the fractions. Since the numerators are both negative, the answer will be a negative number. The numerator of the first fraction (-5) divided by the numerator of the second fraction (5) is -1.

Multiply this answer by the common denominator (18) to get the final answer: -5/2.

learn more about number here

https://brainly.com/question/10547079

#SPJ4

alice wanted to buy 10 markers but was short of $5.20.Then she divided to buy 6 markers and used the remaining 4.40 to buy lunch. how much money did she have at first

Answers

Answer: k

Step-by-step explanation:

tddthvbkj

Let's use algebra to solve this problem. Let x be the amount of money Alice had at first.

If she wanted to buy 10 markers but was short of $5.20, it means the cost of 10 markers was x - 5.20.

Since she decided to buy only 6 markers, she spent 6/10 of the total cost, which is (6/10)*(x - 5.20) = 0.6x - 3.12.

She used the remaining $4.40 to buy lunch, so we can set up an equation:

0.6x - 3.12 + 4.40 = x

Simplifying this equation, we get:

0.4x = 7.52

x = 18.80

Therefore, Alice had $18.80 at first.

Find the perimeter of the shape below:

Answers

Check the picture below.

[tex]~\hfill \stackrel{\textit{\large distance between 2 points}}{d = \sqrt{( x_2- x_1)^2 + ( y_2- y_1)^2}}~\hfill~ \\\\[-0.35em] ~\dotfill\\\\ S(\stackrel{x_1}{-2}~,~\stackrel{y_1}{7})\qquad R(\stackrel{x_2}{-1}~,~\stackrel{y_2}{3}) ~\hfill SR=\sqrt{(~~ -1- (-2)~~)^2 + (~~ 3- 7~~)^2} \\\\\\ ~\hfill SR=\sqrt{( 1 )^2 + ( -4)^2} \implies \boxed{SR=\sqrt{ 17}}[/tex]

[tex]R(\stackrel{x_1}{-1}~,~\stackrel{y_1}{3})\qquad U(\stackrel{x_2}{-1}~,~\stackrel{y_2}{5}) ~\hfill RU=\sqrt{(~~ -1- (-1)~~)^2 + (~~ 5- 3 ~~)^2} \\\\\\ ~\hfill RU=\sqrt{( 0)^2 + ( 2)^2} \implies RU=\sqrt{ 4}\implies \boxed{RU=2} \\\\\\ U(\stackrel{x_1}{-1}~,~\stackrel{y_1}{5})\qquad T(\stackrel{x_2}{2}~,~\stackrel{y_2}{5}) ~\hfill UT=\sqrt{(~~ 2- (-1)~~)^2 + (~~ 5- 5~~)^2} \\\\\\ ~\hfill UT=\sqrt{( 3)^2 + ( 0)^2} \implies UT=\sqrt{ 9}\implies \boxed{UT=3}[/tex]

[tex]T(\stackrel{x_1}{2}~,~\stackrel{y_1}{5})\qquad S(\stackrel{x_2}{-2}~,~\stackrel{y_2}{7}) ~\hfill TS=\sqrt{(~~ -2- 2~~)^2 + (~~ 7- 5~~)^2} \\\\\\ ~\hfill TS=\sqrt{( -4)^2 + ( 2)^2} \implies \boxed{TS=\sqrt{ 20}} \\\\[-0.35em] ~\dotfill\\\\ \stackrel{ \textit{\LARGE Perimeter} }{\sqrt{17}+2+3+\sqrt{20} }~~ \approx ~~ \text{\LARGE 13.6}[/tex]

The sales tax on a $44.00 purchase is $2.42. At this rate, what would be the tax on goods worth $60.00?

Answers

Answer:

We can use proportions to find out the tax on goods worth $60.00:

Let x be the tax on goods worth $60.00.

Then, we can set up the following proportion:

tax/sales = tax rate

or

2.42/44 = x/60

To solve for x, we can cross-multiply and simplify:

44x = 2.42 * 60

44x = 145.20

x = 145.20/44

x = 3.3 (rounded to the nearest cent)

Therefore, the tax on goods worth $60.00 would be $3.30.

Step-by-step explanation:

Suppose we want to estimate the proportion of center party sympathizers with a 95% confidence interval with a statistical margin of error of at most 2% points. How large a sample do we need to take, if we assume that the percentage of centrists is about 6%?

Answers

sample=38

To calculate the size of the sample you need to take in order to estimate the proportion of center party sympathizers with a 95% confidence interval and a statistical margin of error of at most 2% points, you can use the formula n = (Zα/2/E)2 × p × (1-p), where n is the sample size, Zα/2 is the z-score of the desired confidence level (in this case, 1.96 for a 95% confidence interval), E is the margin of error (2%), and p is the population proportion (6%).



Plugging in the values from the question, we get n = (1.96/2)2 × 6% × (1 - 6%) = 38.4. Therefore, the sample size needed to estimate the proportion of center party sympathizers with a 95% confidence interval and a statistical margin of error of at most 2% points is 38.

Learn more about Proportion

brainly.com/question/30657439

#SPJ4

Find the absolute maximum and minimum of the function on the given domain.
f(x,y)=7x^2+2y^2 on the closed triangular plate bounded by the lines x=0, y=0, y+2x=2 in the first quadrant
The absolute maximum is ?
The absolute minimum is ?

Answers

The absolute maximum of the function f(x,y)=7x^2+2y^2 on the given domain is 28 and the absolute minimum is 0.

The absolute maximum and minimum of the function f(x,y)=7x^2+2y^2 on the closed triangular plate bounded by the lines x=0, y=0, y+2x=2 in the first quadrant can be found by using the method of Lagrange multipliers.

First, we need to find the critical points of the function on the interior of the triangular plate. The gradient of the function is given by ∇f = <14x, 4y>. Setting ∇f = 0, we get x = 0 and y = 0. However, these points are on the boundary of the triangular plate, so they are not critical points on the interior.

Next, we need to find the critical points on the boundary of the triangular plate. We can use the method of Lagrange multipliers to do this. The constraint equation is given by g(x,y) = y + 2x - 2 = 0. The gradient of the constraint equation is given by ∇g = <2, 1>. Setting ∇f = λ∇g, we get the following system of equations:

14x = 2λ
4y = λ
y + 2x - 2 = 0

Solving this system of equations, we get two critical points: (2/3, 2/3) and (2, 0).

Finally, we need to evaluate the function at the critical points and at the corners of the triangular plate to find the absolute maximum and minimum. The function values at these points are:

f(0,0) = 0
f(2/3, 2/3) = 14/3
f(2,0) = 28
f(0,2) = 8

The absolute maximum is 28 and the absolute minimum is 0.

Therefore, the absolute maximum of the function on the given domain is 28 and the absolute minimum is 0.

To know more about Lagrange multipliers refer here:

https://brainly.com/question/30776684#

#SPJ11

A new car is purchased for 19700 dollars. The value of the car depreciates at

9. 25% per year. To the nearest year, how long will it be until the value of the

car is 5500 dollars?

Answers

It will take 10.4 years for the value of the car to depreciate to 5500 dollars.

The equation for calculating the number of years until the value of the car is 5500 dollars is:

y = (19700 - 5500) / 0.0925

y = 10.4 years

In order to calculate the number of years until the value of the car is 5500 dollars, we can use the equation y = (19700 - 5500) / 0.0925. This equation uses the initial value of the car (19700) and the desired value (5500), as well as the depreciation rate of 9.25% per year. By solving this equation, we can determine that it will take 10.4 years for the value of the car to depreciate to 5500 dollars.

Learn more about equation here:

https://brainly.com/question/10413253

#SPJ4

1. Four plus a number

2. Twice Daria's age

3. Six times a number plus forty-one

4. The sum of a number and 17

5. The difference between Mary's height and Frank's height

6. The quotient of Iquan's age and 4

7. The product of Arielle's age and 50

8. Seventy-five increased by a number

9. Four hundred decreased by twice a number

Eleven ples more than a number

10.

11. Twice as many dogs

41X

12.

A number doubled plus ten

13. A variable tripled less 40

14. Twice the temperature minus 60 degrees

15. A number divided by fifteen less than 3

16. Five more than a number

17. Thirty-three less than a number

8. Twice Solomon's weight less fifteen pounds

3. The difference between sixty and twice a number

D. The factor of a variable and the coefficient four

Answers

From the given information provided, the given sentences in the form of algebraic expressions are as follows:

1. 4 + x

2. 2D

3. 6n + 41

4. x + 17

5. Mary's height - Frank's height

6. Iquan's age / 4

7. 50Arielle's age

8. 75 + x

9. 400 - 2x

10. 11 + x

11. 2d

12. 2x + 10

13. 3v - 40

14. 2t - 60

15. x/15 - 3

16. 5 + x

17. x - 33

18. 2S - 15

19. 60 - 2x

20. 4D

Question - 1. Four plus a number 2. Twice Dharia's age 3. Six times a number plus forty-one 4. The sum of a number and 17 5. The difference between Mary's height and Frank's height 6. The quotient of Aquaman's age and 4 7. The product of Arielle's age and 50 8. Seventy-five increased by a number 9. Four hundred decreased by twice a number Eleven ples more than a number 10. 11. Twice as many dogs 41X 12. A number doubled plus ten 13. A variable tripled less 40. 14. Twice the temperature minus 60 degrees 15. A number divided by fifteen less than 3 16. Five more than a number 17. Thirty-three less than a number 18. Twice Solomon's weight less fifteen pounds 19. The difference between sixty and twice a number 20. The factor of a variable and the coefficient four. Translate the following into algebraic expression.

Learn more about variable here: brainly.com/question/27894163

#SPJ4

In ΔABC, c = 75 cm,

m∠B=154° and

m∠C=13°. Find the length of a, to the nearest 10th of a centimeter.

Answers

The length of the side a , to the nearest 10th of a centimeter is 75cm

How to determine the value

It is important to note that the sum triangle theorem states that the sum of the interior angles of a triangle is equal to 180 degrees.

Then, we have;

m< A + m< B+ m < C = 180

substitute the values, we have;

m < A = 180 - 154 - 13

subtract the values

m < A = 13 degrees

Using the sine rule, we have that;

sin A/a = sin B/b = sin C/c

Where; the capital letters are the angles and the small letters are the sides.

We have;

sin A/a = sin C/c

substitute the values

sin 13/a = sin 13/75

cross multiply

a = sin 13 × 75/sin 13

a = 75cm

Learn about law of sines at: https://brainly.com/question/27174058

#SPJ1



The girl lifts a painting to a height of 0. 5 m in 0. 75 seconds. How much

power does she use? *

Answers

Answer:

and painting has 100kg or 1kg bcs thats not same

Joshua has a ladder that is 19 ft long. He wants to lean the ladder against a vertical wall so that the top of the ladder is 15 ft above the ground. For safety reasons, he wants the angle the ladder makes with the ground to be no greater than 75°. Will the ladder be safe at this height? Show your work and draw a diagram to support your answer.

Answers

Answer:

50.47°

Step-by-step explanation:

We can use the trigonometric function sine to determine the angle the ladder makes with the ground. Let's call this angle theta. We have:

sin(theta) = opposite/hypotenuse

where the opposite side is the height of the ladder on the wall (15 ft) and the hypotenuse is the length of the ladder (19 ft). Solving for theta, we get:

theta = sin^-1(15/19) ≈ 50.47°

Since this angle is less than 75°, the ladder will be safe at this height.

2.5 m³ of limestone has a mass of 6025 kg. a) Calculate the density of limestone in kg/m³. 3 b) Find the mass of 1.7 m³ of limestone in kg.​

Answers

the mass of 1.7 m³ of limestone is 4097 kg.

The soccer team plays every 4 days and the basketball team plays every 5 days. When will both teams have games on the same day again?

Answers

Answer:

Step-by-step explanation:

The soccer team:

1 2 3 4, 1 2 3 4, 1 2 3 4, 1 2 3 4, 1 2 3 4

The basketball team:

1 2 3 4 5, 1 2 3 4 5, 1 2 3 4 5, 1 2 3 4 5.

[ the bold number is the day of playing ]

Hope this helps.

Last month Carmen made $480 working for 30 hours this month she will get a 15% increase in the amount she earns per hour what will be her hourly rate in dollars after the increase enter your answer in the space provided

Answers

After the 15% increase in her hourly rate, Carmen's new hourly rate will be $18.40 per hour.

Carmen currently makes $480 in a month by working 30 hours. To find her hourly rate, we can divide her total earnings by the number of hours she worked:

Hourly rate = Total earnings ÷ Number of hours worked

So Carmen's current hourly rate is:

Hourly rate = $480 ÷ 30 = $16 per hour

If Carmen gets a 15% increase in her hourly rate, we can calculate her new hourly rate by multiplying her current hourly rate by 1.15 (since a 15% increase means the new rate is 115% of the current rate):

New hourly rate = Current hourly rate x 1.15

New hourly rate = $16 x 1.15

New hourly rate = $18.40 per hour

This means she will earn $2.40 more per hour than she did before the increase.

To learn more about hourly rate click on,

https://brainly.com/question/21186333

#SPJ4

A large company put out an advertisement in a magazine for a job opening. The first day the magazine was published the company got 125 responses, but the responses were declining by 24% each day. Assuming the pattern continued, how many total responses would the company get over the course of the first 8 days after the magazine was published, to the nearest whole number?​

Answers

18 responses would the company get over the course of the first 8 days after the magazine was published.

What is a geometric sequence?

A geometric progression, often referred to as a geometric sequence, is a series of non-zero values where each term following the first is obtained by multiplying the preceding value by a constant, non-zero number known as the common ratio.

Here, we have

Given: A large company put out an advertisement in a magazine for a job opening. On the first day, the magazine was published the company got 125 responses, but the responses were declining by 24% each day.

We apply here geometric sequence.

aₙ = arⁿ⁻¹

where

aₙ = n^{th} term of the sequence

r = is the common ratio

a = the first term of the sequence

a = 125

r = 100% - 24% = 76% = 76/100 = 0.76

aₙ = (125)(0.76)⁸⁻¹

aₙ = 125(0.76)⁷

aₙ = 18

Hence, 18 responses would the company get over the course of the first 8 days after the magazine was published.

To learn more about the geometric sequence from the given link

https://brainly.com/question/24643676

#SPJ1

Answer:

463 (to the nearest whole number)

Step-by-step explanation:

We can model the given scenario as a geometric sequence.

The first term, a, is the number of responses the company got on the first day:

a = 125

The common ratio is the number you multiply by at each stage of the sequence. As the responses are declining by 24% each day, then each day the responses are 76% of the previous day's responses, since 100% - 24% = 76%. Therefore, the common ratio, r, is:

r = 0.76

To calculate the total responses the company would get over the course of the first 8 days after the magazine was published, use the Geometric Series formula.

[tex]\boxed{\begin{minipage}{7 cm}\underline{Sum of the first $n$ terms of a geometric series}\\\\$S_n=\dfrac{a(1-r^n)}{1-r}$\\\\where:\\\phantom{ww}$\bullet$ $a$ is the first term. \\ \phantom{ww}$\bullet$ $r$ is the common ratio.\\\end{minipage}}[/tex]

Substitute a = 125, r = 0.76 and n = 8 into the formula and solve for S:

[tex]\implies S_8=\dfrac{125(1-0.76^8)}{1-0.76}[/tex]

[tex]\implies S_8=\dfrac{125(1-0.111303478...)}{0.24}[/tex]

[tex]\implies S_8=\dfrac{125(0.888696521...)}{0.24}[/tex]

[tex]\implies S_8=\dfrac{111.087065...}{0.24}[/tex]

[tex]\implies S_8=462.862771...[/tex]

[tex]\implies S_8=463[/tex]

Therefore, the total number of responses the company would get over the course of the first 8 days after the magazine was published is 463 to the nearest whole number.

find number of conversion periods and rate of interest when compounded half-yearly for a sum of Rs 5000 is taken for 7 years and 9% p.a

Answers

The rate of interest when compounded half-yearly is 4.45% and there are 14 conversion periods.

Solving compounded interest

The formula for calculating the compound interest is:

A = P (1 + r/n)^(n*t)

Where:

A = Final amountP = Principal amountr = Annual interest rate (as a decimal)n = Number of times the interest is compounded per yeart = Time period (in years)

In this case, P = Rs 5000, r = 9% p.a. and the interest is compounded half-yearly (i.e., n = 2).

To find the number of conversion periods, we need to multiply the number of years by the number of conversion periods per year:

Number of conversion periods = n*t = 2 * 7 = 14

So there are 14 conversion periods in 7 years.

To find the rate of interest when compounded half-yearly, we can rearrange the formula and solve for r:

A = P (1 + r/n)^(nt)

A/P = (1 + r/n)^(nt)

(1 + r/n) = (A/P)^(1/nt)

r/n = (A/P)^(1/nt) - 1

r = n[(A/P)^(1/n*t) - 1]

Substituting the given values, we get:

r = 2[(5000*(1 + 0.09/2)^(27))^(1/(27)) - 1]

= 0.0445 or 4.45%

Therefore, the rate of interest when compounded half-yearly is 4.45%.

Learn more on compound interest here: https://brainly.com/question/24924853

#SPJ1

Please help me answer my homework in the image

Answers

Answer:

4 bc if u go by the side where a block counts u count it till that side ends and it will be 4

Units of Capacity
Customary
System Units
1 gallon
1 quart
1 cup
Metric System Units
3.79 liters
0.95 liters
0.237 liters
Sameer usually drinks 3 cups of coffee in the morning.
How many liters of coffee does he drink? Round your
answer to the nearest tenth.
3 cups
1
X
new units
original units
Sameer drinks
morning.
=?
=
liters of coffee in the

Answers

As given the conversion unit: 1 cup = 0.237 liters. Sameer drinks 0.71 liters of coffee.

Explain about the units of measurements?

Any physical quantity can be measured by comparing it to a recognised standard, and the magnitude is almost always expressed in terms of the reference standard known as a unit.

The FPS system, which measures length, mass, plus time in feet, pounds, and seconds, is one of the three systems that also was utilised for the measurement. The MKS system, which stands for metre, kilogramme, and seconds, has replaced the CGS system in centimetre, gramme, and seconds as the one that is widely used.

Units of Capacity are given as:

System Units      Metric System Units

1 gallon       -       3.79 liters

1 quart        -       0.95 liters

1 cup           -       0.237 liters

Coffee Intake of Sameer:  3 cups

1 cup           -       0.237 liters

Multiply both sides by 3.

1*3 cup           -       0.237*3 liters

3 cup           -        0.711  liters

Thus, Sameer drinks 0.71 liters of coffee (rounded off nearest tenth).

Know more about the units of measurements

https://brainly.com/question/777464

#SPJ1

Complete question:

Units of Capacity are given as:

System Units      Metric System Units

1 gallon       -       3.79 liters

1 quart        -       0.95 liters

1 cup           -       0.237 liters

Sameer usually drinks 3 cups of coffee in the morning. How many liters of coffee does he drink? Round your answer to the nearest tenth.

Questions are in the following picture

Answers

1. If there were two people dividing the cost of the gift, each person would spend $180. If there were three people dividing the cost, each person would spend $120. If there were five people dividing the cost, each person would spend $72. If there were ten people dividing the cost, each person would spend $36. If there were one hundred people dividing the cost, each person would spend $3.60.

2. The function that could be used to model the amount each person would spend depending on the number of people contributing to the gift is:

cost per person = total cost / number of people

3. The table will be:

Number of people Amount per person

2. $180

3 $120

5. $72

10. $36

100. $3.60

The graph of the function would be a straight line passing through the points (2, $180), (3, $120), (5, $72), (10, $36), and (100, $3.60).

4. The domain of the function is all positive integers greater than zero, since you cannot have a fractional or negative number of people contributing.

How to explain the information

The range of the function is all positive real numbers, since the cost per person can be any positive amount.

The function is decreasing, since the cost per person decreases as the number of people contributing increases.

There is no maximum or minimum value for the cost per person, since it can be any positive amount. The function is continuous, but the number of people contributing must be a discrete value (i.e., a whole number).

As the number of people contributing approaches infinity, the cost per person approaches zero. The y-intercept of the function is the cost of the gift, and the x-intercept is not applicable in this context.

There is a horizontal asymptote at y = 0, since the cost per person approaches zero as the number of people contributing approaches infinity.

Learn more about functions on:

https://brainly.com/question/10439235

#SPJ1

Chile is celebrating her Quinceañera. Hannah knows the perfect gift to buy Chile, but it costs $360. Hannah can't afford to pay for this on her own so thinks about asking some friends to join in and share the cost.

1. How much would each person spend if there were two people dividing the cost of the gift? How much would each person spend if there were three people dividing the cost? Five people? Ten? One hundred?

2. Determine the function that could be used to model the amount each person would spend depending on the number of people contributing to the gift.

3. Use multiple representations to show how the amount each person would contribute to the gift would change depending on the number of people contributing. Describe the connections between the representations.

4. Describe the features of the function based on the context (domain/range, increasing/decreasing, maxima/minima, discrete/continuous, end behavior, intercepts, asymptotes).

Test Prep: Simplify −5 (7 + x) + 2 5/6x.

Answers

Answer:

-35 - 5/6x

Step-by-step explanation:

Other Questions
Lumber is a renewable and self-maintaining resource. It comes from the cutting down of different types of trees. For example, lumber from pine trees is often used for making paper. Which of the following processes enables lumber to be a self-maintaining renewable resource? A. the process of decomposition B. the process of manufacturing C. the process of natural selection D. the process of photosynthesis 1. What are two critical variables that have affected the Russian economy in recent years? Checkall that apply:a. Economic sanctions due to their annexation of Crimeab.Global warmingc. Plunging oil pricesd. Intervention in Syria Find the length of the missing side. Find m angle1 and m angle2 How did the Allies take control of Italy?A. By invading France to divert Nazi troops away from ItalyB. By invading in the south at Sicily and advancing north to RomeC. By bombing Rome day and night until the Italian army surrenderedOD. By trapping German forces in northern Italy in the winter with nofoodSUBMIT 2NaClO3 2NaCl + 3O2How many moles of oxygen gas are produced when 7.3 moles of sodium chlorate, NaClO3, decomposes? A reaction conducted in a calorimeter consumed 0.038mol of an unknown substance. The temperature of the 109g of water was increased from 13.2 to 25.7. What is the enthalpy of this reaction in kJ/mol? Percentage16% of 350 motivational research is based on the trait theory of personality. true/false Using Coral Pseudocode, design the logic for a program that uses arrays based on the following scenario:Trainers at Coyote Athletic Club are encouraged to enroll new members. Write an application that allows the owner Wile E. Coyote to enter the number of new members each trainer has enrolled this year. The number of trainers will vary each year so make sure you enable it to allow Wile to select how many trainers he has before entering the number of members enrolled for each trainer in an array. Add up all the members that fall within the correct range into an array before outputting the totals.0 - 5 members6 - 12 members 13 - 20 membersMore than 20 membersExample Input for TrainersValue Entered for Number of Trainers: 10Values Entered Member Enrolled: 2 13 7 13 20 0 15 21 5 25 My names Sara. I (1) love (love) skateboarding Im a real fan. I (2) (practise) at a local park every weekend. I (3) (not often/do) competitions because Im from a small town. Im very excited today because I (4) (perform) in a music video for Way In. At the moment were (5) (get) ready. Lots of people (6) (come) into the park now. My mum and dad (7) (sit) near the front because they (8) (want) to take photos and upload them for their friends! name four ways how self awareness can decrease the likelihood of learners turning to discriminating behaviors Identify the four elements of entrepreneurship, and how they come together and support each other over time. Identify the four elements of entrepreneurship, and how they come together and support each other over time. If you begin with a 400 gram sample of Rubidium-87 and only 100 gramsremainHow many half-lives have gone by?How old is the sample? 8) For a given area of a triangle, the base varies inversely as its height. When the height is10 in the base is 5 in. Find the base if the height is increased to 20 in. 400-kg go-cart begins at rest and obtains a velocity of 20 m/s east after 10 seconds. a.) What is the objects initial momentum? 2. 1 define the term study plan and state four ways in which stress may negatively affect your ability to answer questions in an examination session Investigamos sobre las estrategias para redactar textos y sobre las propiedades textuales Indicate whether the sentence or statement is true or false.Many women in the West were granted the right to vote.Please select the best answer from the choices providedOTOF The headlights of an automobile are set such that the beam drops 2. 00 in. For each 20. 0 ft in front of the car. What is the angle between the beam and the road?The angle between the beam and the road is. _____(Round to one decimal place as needed. )